Subtract and write your answer as a mixed number 9 4/5 - 7 1/2

Answers

Answer 1
The answer is 2 3/10

Related Questions

AGAIN LAST BUT NOT LEAST
This is for the smarty pants
and the person knows who I am talking bout

Answers

Answer:

Mr. Smarty Pants here

Distance between some points

A and B = 2

B and C = 5

C and B = 5

Four points that are 7 away from another point

-2,1

5,-6

5,8

12,1

Distance between some points

4

$10.00/8 slices is this a unit rate .

Answers

Answer:

No

Step-by-step explanation:

Unit rate consists of at least one 1. the unit rate for this would be 1 slice for $1.25

please mark brainliest

¿cual es la raíz cuadrada de √4565?

Answers

Answer:

8.21978002908

Chloe works in a department store selling clothing. She makes a guaranteed salary of $250 per week, but is paid a commision on top of her base salary equal to 10% of her total sales for the week. How much would Chloe make in a week in which she made $975 in sales? How much would Chloe make in a week if she made xx dollars in sales?

Answers

Answer:

See Below

Step-by-step explanation:

Making $250/week in salary and 10% of the $975 in sales.

First find what 10% of $975 equals.  The decimal equivalent of 10% is 0.10

So multiply the decimal and total sales.

0.10 x 975 = 97.5

Her weekly salary plus the commission would be:

$250 + $97.50 = 347.50

If you don't know her sales for the week, you can use X to represent the sales.  The equation would be:

0.10x   +   250  =  y       where y is her total pay.

Hey , May someone help me please and thanks<3

Answers

Answer:

[tex]1\frac{1}{5}[/tex]

Step-by-step explanation:

6-4 = 2

2 -  [tex]\frac{4}{5}[/tex]  = [tex]1\frac{1}{5}[/tex]

Please help me answer this

Answers

The answer to that equation if x = -2 would be 41

Answer:

→ 9x^2 - 4x - 3 = [41].

Step-by-step explanation:

Simplify:

9x^2 - 4x -3

=> 9(-2)^2 - 4(-2) - 3 [ x = 1 (Given)]

=> 9(4) - 4(-2) - 3

=> 36 + 8 - 3

=> 44 - 3

=> 41 Ans.

Stanley ran 48 of a mile. Gary ran 36 of a mile. Who ran farther

Answers

stanley ran farther then gary ran

A polynomial is a factor of another polynomial if when divided they have a remainder of zero. True or False.

Answers

Answer:

True

Step-by-step explanation:

The Factor theorem states that  for any polynomial f(x) if f(c)=0 then x-c is a factor of the polynomial f(x).

If any polynomial f(x) is divided by x-a and remainder is 0 that means f(a)= 0 .In other words we can say x-a is a factor of the polynomial f(x).So the statement :If a polynomial is divided by (x-a) and the remainder equals zero then (x -a) is a factor of the polynomial is True.

Have a happy day! <3Miss Hawaii

find the measure of each angle plz do 15 and 16

Answers

Answer:

15) 55°

16) 90°

Step-by-step explanation:

15)

55° + 75° + x = 180°

x = 50°

___

50° + 60° + x = 180°

x = 70°

__

70° + 55° + ? = 180°

? = 55°

______________________________

16)

55° + 90° + x = 180°

x = 35°

__

35° + x = 90°

x = 55°

___

55° + 35° + ? = 180°

? = 90°

Please help! I’ll give brianliest to the correct answer!

Answers

Answer:

y = 1.5x + 9  

(y = height; x = number of cups)

Step-by-step explanation:

let x = number of cups

let y = height of the stack

the first cup will add 9 to the height (7.5 + 1.5)

so 9 is the y-intercept

for each additional cup, 1.5 will be added

so y = 1.5x + 9

Select all the statements that best describe the graph below.

Answers

Answer:

the independant variable is x

The Correct choices are :

dependent variable is y

rate of change is constant

this is a negative relationship

the independent variable is x

the relationship is linear

Which of the following shows the factors of the polynomial f (x) = x^3 -11x + 6 , if one of those factors is x - 3

a. x - 3 is not a factor and there are no correct answers
b. f(x) = (x -3) ( x^2 - 8x - 18)
c. f(x) = (x -3) ( x^2 + 3x - 2)
d. f(x) = (x -3) ( x^2 - 3x^2 - 2x+12)

Answers

Answer: f(x)=(x-3)(x^2+3x-2)

Step-by-step explanation:

Approximately how much greater, in grams is 191520 than 146640

Answers

191520 grams is greater than 146640 grams by 44,880 grams

what set of ordered pairs in the form of (x,y) does not represent a function of x

Answers

Answer:

I believe it's option C

Step-by-step explanation:

A function is a relation that does not contain two pairs with same first component.

option C

{(-1,2),(3,-2),(0,1),(3,5)} is not a function, since there are two pairs with the same first component i.e, 3

hope it helps

mark me brainliest pls

Complete straight line graph using a table

Answers

Answer:

x as -3 = y as 2

x as 0 = y as 3

x as 3 = y as 4

x as 6 = y as 5

Step-by-step explanation:

To answer what the output (y) is equal to, all you do is replace the x with whatever value you are trying to find the output to.

subtract from 15 the sum of 2⅓ and 4⅖​

Answers

Answer:

-8 4/15

Step-by-step explanation:

[tex]2\frac{1}{3} + 4\frac{2}{5} -15[/tex] [tex]\frac{7}{3} +\frac{22}{5} -15[/tex] [tex]\frac{35}{15} + \frac{66}{15} - 15[/tex] [tex]\frac{101}{15} - 15[/tex] [tex]6\frac{11}{15} - 15[/tex] [tex]\frac{-124}{15}[/tex] [tex]-8 \frac{4}{15}[/tex]

What would be the most common example of a curve with constant widith

Answers

Answer:

a circle

Step-by-step explanation:

yeah-ya... right?

PLEASE HELP ME !!!!!! EASY!!!!

Answers

Answer:

JL = √38.

a = 1 and b = 38.

Step-by-step explanation:

Using Pythagoras:

(JL)^2 = (√8)^2 + (√30)^2

(JL)^2 = 8 + 30 = 38.

JL = √38.

2.75 mi+3 mi+ 2.5 mi =?

Answers

Answer: 8.25mi

Step-by-step explanation:

Just add them all together

8.25 mi

Step-by-step explanation:

2.75 mi + 3 mi + 2.5 mi

= (2.75 + 3 + 2.5)mi

= (5.75 + 2.50)mi

= 8.25 mi

Jane started jogging home from 5 miles away, at a rate of 2 mph. Which equation in slope-intercept form represents Jane's distance from home?

Answers

Answer:

10 miles

Step-by-step explanation:

Solve:

e + 13.2 = 289.47

Answers

Answer:

Evaluate.

15.91828182 = 289.47

Help !! Multiply: 2w(w+17)

Answers

Answer:

2w^2 + 34w

Step-by-step explanation:

2w(w+17)

Multiply the 2w by each term in the parentheses

2w*w + 2w*17

2w^2 + 34w

will give brainliest

Which expression is equal to 27√ ?

147√

27√49

27√7

727√

Answers

3/3 your welcome it’s the expression

Please help me out I really can’t afford failing this test

Answers

Answer:

I'm not really sure but I think it's 2

Step-by-step explanation:

So here's what I did;

x³+3x-9=x - 1 +2x

x³+3x - x -2x = 9 - 1

x³= 8

x = ³√8

x = 2

I hope this helps

Solve for m < LMN.......

Answers

Answer:

m< LMN = 111°

Step-by-step explanation:

m< LMO + m<OMN = m< LMN

[tex]58 + 53 = 111[/tex]

-3x lessthan or equal to 3x +3 less than 2x+ 10

Answers

Answer:

x ≥ 0Hope this helped :)

6
of a yard on the following play. How many more yards did the
5
One football player advances
8
of a yard on a play. Another football player advances
second player advance than the first player?
А
13
yards
56
B
C с
21
yards
1
yards
7
10
15 yards
D
MO 15
O
11:29

Answers

Answer:

Step-by-step explanation:

B=brackets

i= indices

D= division

M=multiplication

A= addition

S= subtraction

7x10000/5x100/6x1

7x10000=70000/500/6

70000/500=140/6= 23.33

the ratio of a shorter walkway to a longer walkway is 3 to 5. the lenght of the sorter walkway is 45 meters. what is the lenght of the longer walkway

Answers

Answer:

75 m

Step-by-step explanation:

Let s- shorter walkway, l- longer walkway.

Given:

s/l = 3/5s = 45 m

Find the length of l:

45/l = 3/5l = 45*5/3l = 75 m

Answer:

[tex]\sf\longmapsto \: length = 75 \: metres[/tex]

Step-by-step explanation:

Now let's find out what's the length of the long walkway–

[tex]\sf\longmapsto \: \frac{3}{5} = \frac{45}{\: length\: of \:longer\: walkway} [/tex]

[tex]\sf\longmapsto \: \: length\: of \:longer\: walkway= \frac{45 \times \: 5 }{3} [/tex]

[tex]\sf\longmapsto \: length\: of \:longer\: walkway= 75 \: metres[/tex]

The length of the longer walkway is [tex]\sf\longmapsto \: length = 75 \: metres[/tex]

What is an algebraic expression for this word phrase?

5 more than the product of 7 and n


5 + 7n

5(7+n)

7(5+n)

5 + 7 + n

Answers

Answer:

The value of the algebraic expression when n is given as 6 will be 38.

An algebraic expression is when we use numbers and words in solving a particular mathematical question.

Based on the information given in the question, the expression will be:

= 1 + 7n - 5

Note that the value of n = 6

Therefore, 1 + 7n - 5 will be:

= 1 + 7n - 5

= 1 + 7(6) - 5

= 1 + 42 - 5

= 38

Step-by-step explanation:Therefore, the value of the algebraic expression when n is given as 6 is 38.

numbers between 1/10 and 4/5?​

Answers

I believe your answer is 2/10,3/10,4/10,5/10,6/10,7/10.

4/5 can be converted to 8/10 by multiplying the denominator and the numerator by 2 so the new numbers are 1/10 and 8/10 the numbers that are between that are 2/10,3/10,4/10,5/10,6/10,7/10.

Other Questions
Which is written correctly?A)Amber, wants to be a hospital volunteerB)Amber wants to be a hospital volunteerC)Amber wants to be, a hospital volunteerD)Amber wants to be a hospital volunteer ?????????????????????????????????????? let f(x)=-2x-7 and g(x)=-4x+3 find (fg)(-5) It isnt safe to go out today. There was too much S2 yesterday. Many streets are flooded with wate simplify the expression 8xy-(x+2xy)+3x Which describes a molecule? (Select all that apply.)It can be one element by itself.It can be the combination of different neutrons.It can be two or more different elements combined.It can be two or more of the same elements combined together. please help! What is the measure of angle 1? Put the number only. Which one of these Amendments: 16th,17th, 18th, or 19th do you think has been the most important for the American people since it was passed? Why? someone help me please :3 help me please i really need this right now MOST IMPORTANT AMENDMENT" and in a 4-5 sentence paragraph, make an argument for which amendment is most important and why? Do you believe that there is still an effort to undermine the voting rights of people ofcolor? Why or why not? What's is the meaning of the word chivalry ? which type of blood vessel holds the greatest volume of blood? If y=cosh(4tanhx) then dy/dx= you are approaching an intersection on a multiple-lane road, and you want to change lanes; In may, Ana made one deposit of $200 to her savings account and made 3 withdrawals of $50 each. How much money did she have in her account at the end of may?Make an equation with integer and than answer: What is x+y=5 solved and graphed????I need help, please! Find the area of the triangle.68 A car is traveling at 40 mi/hour. What is the car's speed in feet per second? help me I beg you please